Archivo del Autor: Juan Manuel Naranjo Jurado

Cálculo Diferencial e Integral I: El número de Euler

Por Juan Manuel Naranjo Jurado

Introducción

Supongamos que tenemos un peso el cual hemos decidido invertir y, para hacerlo, tenemos tres opciones. La primera es invertir en el Banco A que está dispuesto a regresarnos un 100% de interés después de un año. En este caso, al finalizar el año tendríamos el doble de dinero que con el que iniciamos, es decir, tendríamos $ \$2.00.$

Nuestra segunda opción es el Banco B, que nos propone invertir nuestro dinero con ellos y promete generar el 50% del capital dos veces al año. Asumiendo que reinvertimos el dinero obtenido de tal forma que el capital inicial del segundo semestre es igual al capital final del primero, tendríamos lo siguiente

SemestreInterésCapital inicialCálculoCapital final
150% / $\frac{1}{2}$$1.00$1 \cdot (1+\frac{1}{2})$$1.50
250% / $\frac{1}{2}$$1.50$1.5\cdot (1+\frac{1}{2})$$2.25

Notemos que el cálculo podemos hacerlo de forma directa mediante la siguiente expresión

$$\left(1 \cdot \left(1+\frac{1}{2} \right) \right) \cdot \left(1+\frac{1}{2} \right) = 1 \cdot \left(1+\frac{1}{2} \right)^2.$$

Por otro lado, la tercera opción, el Banco C, promete entregarnos 25% de interés cada trimestre. Si después de cada trimestre se invierte todo el capital inicial más los intereses generados, tenemos el siguiente escenario.

TrimestreInterésCapital inicialCálculoCapital final
125% / $\frac{1}{4}$$1.00$1 \cdot (1+\frac{1}{4})$$1.25
225% / $\frac{1}{4}$$1.25$1.25 \cdot (1+\frac{1}{4})$$1.5625
325% / $\frac{1}{4}$$1.5625$1.5625 \cdot (1+\frac{1}{4})$$1.953125
425% / $\frac{1}{4}$$1.953125$1.953125 \cdot (1+\frac{1}{4})$$2.441406

De igual forma, podemos compactar los cálculos anteriores:

$$1 \cdot \left(1+\frac{1}{4} \right) \cdot \left(1+\frac{1}{4} \right) \cdot \left(1+\frac{1}{4} \right) \cdot \left(1+\frac{1}{4} \right) = 1 \cdot \left(1+\frac{1}{4} \right)^4.$$

Después de analizar todas las opciones, vemos que el Banco C nos permite generar más dinero al final del año siendo la mejor de nuestras tres opciones. Una interrogante natural después de haber evaluado los ejercicios anteriores es saber qué sucede si tenemos una tasa de interés de $\frac{1}{365}$ de forma diaria, lo cual generaría

$$1 \cdot \left(1 + \frac{1}{365} \right)^{365} = 2.714567.$$

Este escenario nos permite ganar una mayor cantidad de dinero al final del periodo. ¿Qué pasaría si tuviésemos una tasa de interés que se paga cada hora o cada minuto o cada segundo? Con una periodicidad lo suficientemente alta, ¿podríamos hacernos infinitamente ricos? Esta última pregunta la responderemos analizando el siguiente límite:

$$\lim_{n \to \infty} \left( 1 + \frac{1}{n} \right)^n.$$

El número de Euler

Después de la motivación dada en la introducción, definiremos la sucesión $\{e_n\}$ tal que $e_n = \left( 1 + \frac{1}{n} \right).$

Por los ejemplos revisados donde se calcula el interés en diferentes periodicidades, podemos inferir que la sucesión $\{e_n\}$ es creciente y a continuación lo probaremos.

Proposición. La sucesión $\{e_n\}$ es creciente.

Demostración.

Usando la fórmula del binomio de Newton se tiene lo siguiente

\begin{align*}
e_n & = \left( 1 + \frac{1}{n} \right)^n \\ \\
& = \sum_{ k = 0}^{n} { n \choose k } \frac{1}{n^k} \\ \\
& = \sum_{ k = 0}^{n} \frac{1}{k!} \cdot \frac{n (n-1) \ldots (n-k+1)}{n^k} \\ \\
& = \sum_{ k = 0}^{n} \frac{1}{k!} \cdot \frac{n}{n} \cdot \frac{n-1}{n} \ldots \frac{n-k+1}{n} \\ \\
& = \sum_{ k = 0}^{n} \frac{1}{k!} \cdot 1 \cdot \left( 1 – \frac{1}{n} \right) \ldots \left( 1 – \frac{k-1}{n} \right) \tag{1} \\ \\
& = 1 + 1 + \frac{1}{2!} \left(1 – \frac{1}{n} \right) + \frac{1}{3!} \left(1 – \frac{1}{n} \right) \left(1 – \frac{2}{n} \right) + \ldots + \frac{1}{n!} \left( 1 – \frac{1}{n} \right) \left( 1 – \frac{2}{n} \right) \ldots \left( 1 – \frac{n-1}{n} \right).
\end{align*}

$$\therefore e_n = 1 + 1 + \frac{1}{2!} \left(1 – \frac{1}{n} \right) + \frac{1}{3!} \left(1 – \frac{1}{n} \right) \left(1 – \frac{2}{n} \right) + \ldots + \frac{1}{n!} \left( 1 – \frac{1}{n} \right) \cdot \left( 1 – \frac{2}{n} \right) \ldots \left( 1 – \frac{n-1}{n} \right).$$

Análogamente, se tiene que

\begin{align*}
e_{n + 1} = & 1 + 1 + \frac{1}{2!} \left(1 – \frac{1}{n+1} \right) + \frac{1}{3!} \left(1 – \frac{1}{n+1} \right) \left(1 – \frac{2}{n+1} \right) + \ldots \\ \\
& + \frac{1}{n!} \left( 1 – \frac{1}{n+1} \right) \cdot \left( 1 – \frac{2}{n +1} \right) \ldots \left( 1 – \frac{n-1}{n+1} \right) \\ \\
& + \frac{1}{(n+1)!} \left( 1- \frac{1}{n+1} \right) \left( 1 – \frac{2}{n+1} \right) \ldots \left( 1 – \frac{n}{n+1} \right).
\end{align*}

Notemos que

$$\left( 1 – \frac{1}{n} \right) < \left( 1 – \frac{1}{n+1} \right), \ldots, \left( 1- \frac{k-1}{n} \right) < \left( 1- \frac{k-1}{n+1} \right).$$

Es decir, cada término de $e_n$ es más chico que su correspondiente de $e_{n+1}$ a partir del tercero. Además, $e_{n+1}$ tiene un término positivo extra, se sigue entonces que $e_n < e_{n+1}.$

Por tanto, $\{e_n\}$ es creciente.

$\square$

Antes de continuar, probaremos una proposición respecto a la serie geométrica.

Proposición. Sea $r \in \mathbb{R}$ tal que $|r|<1$, entonces

$$\sum_{k=0}^{n} r^k = \frac{1-r^{n+1}}{1-r}.$$

Demostración.

Sea $S_n = 1 + r + r^2 + r^3 + \ldots + r^n$. Entonces se tiene que

$$r \cdot S_n = r + r^2+r^3+r^4+\ldots+r^{n+1}.$$

Restando $S_n-rS_n$ se tiene

\begin{align*}
S_n-rS_n & = 1 + r + r^2 + r^3 + \ldots + r^n-(r + r^2+r^3+r^4+\ldots+r^{n+1}) \\
& = 1-r^{n+1}.
\end{align*}

\begin{gather*}
\Rightarrow & S_n-rS_n = 1-r^{n+1}. \\
\Rightarrow & S_n (1-r) = 1-r^{n+1}.
\end{gather*}

$$\therefore S_n = \frac{1-r^{n+1}}{1-r}.$$

$\square$

Al inicio se planteó la siguiente pregunta: con una periodicidad lo suficientemente alta, ¿podríamos hacernos infinitamente ricos? La respuesta es no y lo probamos en la siguiente proposición.

Proposición. La sucesión $\{e_n\}$ está acotada entre 2 y 3.

Demostración.

Dado que $\{ e_n\}$ es creciente, se tiene que $e_1 \leq e_n$ para todo $n \in \mathbb{N}$. Así, $e_1 = \left( 1 + \frac{1}{1} \right)^{1} = 2$. Por tanto, $2 \leq e_n$ para todo $n \in \mathbb{N}.$

Ahora probaremos que $3$ es una cota superior de la sucesión.

Notemos que $k! = 1 \cdot 2 \cdot 3 \ldots k \geq 1 \cdot 2 \cdot 2 \ldots 2 = 2^{k-1}$ y de $(1)$ tenemos que

\begin{align*}
e_n & = \sum_{k = 0}^{n} \frac{1}{k!} \cdot 1 \cdot \left( 1 – \frac{1}{n} \right) \ldots \left( 1 – \frac{k-1}{n} \right) \\ \\
& < \sum_{k = 0}^{n} \frac{1}{k!} \\ \\
& = 1 + \sum_{k = 1}^{n} \frac{1}{k!} \\ \\ 
& \leq 1 + \sum_{k = 1}^{n} \frac{1}{2^{k-1}} \\ \\
& = 1 + \sum_{k = 0}^{n-1} \frac{1}{2^{k}}.
\end{align*}

Además, por la proposición anterior se tiene que

$$ \sum_{k = 0}^{n-1} \frac{1}{2^k} = \frac{1 – \frac{1}{2^n}}{1- \frac{1}{2}} <2.$$

Por tanto, se sigue que $$1 + \sum_{k = 0}^{n-1} \frac{1}{2^{k}} < 3.$$

Se concluye que $2 < e_n < 3$.

$\square$

Hemos probado que $\{e_n\}$ es una sucesión creciente y acotada. Por tanto, se sigue que es convergente, y converge al supremo. Definimos el número de Euler de la siguiente forma:

$$e := \lim_{n \to \infty} \left( 1 + \frac{1}{n} \right)^n.$$

A continuación, mostramos la gráfica de esta sucesión.

Este número ha logrado posicionarse como uno de los más conocidos dentro del mundo de las matemáticas, siendo $e$ la base del logaritmo natural y dentro de este mismo curso se ha usado antes al momento de estudiar la función exponencial.

Más adelante…

En esta unidad hemos revisado a detalle el límite de una sucesión y en la unidad anterior estudiamos el concepto y propiedades de las funciones, es tiempo de continuar con un concepto más avanzado que requiere del entendimiento de ambos temas: el límite de una función.

Tarea moral

A continuación hay algunos ejercicios para que practiques los conceptos vistos en esta entrada. Te será de mucha utilidad intentarlos para entender más la teoría vista.

Encuentra los siguientes límites:

  • $$\lim_{n \to \infty} \left(1+\frac{1}{n}\right)^{n+1}.$$
  • $$\lim_{n \to \infty} \left(1+\frac{1}{n+1}\right)^n.$$
  • $$\lim_{n \to \infty} \left(1+\frac{1}{n}\right)^{2n}.$$
  • $$\lim_{n \to \infty} \left(1-\frac{1}{n}\right)^n.$$

Entradas relacionadas

Agradecimientos

Trabajo realizado con el apoyo del Programa UNAM-DGAPA-PAPIME PE104522 «Hacia una modalidad a distancia de la Licenciatura en Matemáticas de la FC-UNAM – Etapa 2»

Cálculo Diferencial e Integral I: Sucesiones de Cauchy

Por Juan Manuel Naranjo Jurado

Introducción

En las entradas anteriores vimos las propiedades de una sucesión convergente para lo cual era necesario conocer su límite. En esta ocasión estudiaremos a las sucesiones de Cauchy, éstas cumplen una propiedad particular: dado un valor positivo arbitrario, existe un momento a partir del cual la distancia entre dos términos cualesquiera de la sucesión es menor al valor arbitrario establecido. Además, probaremos la relación entre este tipo de sucesiones y las sucesiones convergentes.

Sucesiones de Cauchy

La definición formal de sucesión de Cauchy se da a continuación.

Definición. Se dice que una sucesión $\{a_n\}$ de números reales es una sucesión de Cauchy si para todo $\varepsilon > 0$ existe un número natural $k$ tal que para todos los números naturales $n$, $m \geq k$ se satisface que $|a_n – a_m| < \varepsilon.$

En esta entrada demostraremos la equivalencia entre que una sucesión sea de Cauchy y que sea convergente. La gran ventaja que presenta el concepto de sucesión de Cauchy es que podremos probar que una sucesión converge sin necesidad de conocer su límite, puesto que la definición no hace uso de él. Pero antes de probarlo, veremos un par de ejemplos para familiarizarnos con la definición.

Ejemplo 1. La sucesión $\{\frac{1}{n}\}$ es una sucesión de Cauchy.

Demostración.

Sea $\varepsilon > 0$.

Tomemos $k > \frac{2}{\varepsilon}$. Si $n$, $m > k$, entonces $\frac{1}{n} < \frac{1}{k} < \frac{\varepsilon}{2}$. Análogamente se tiene que $\frac{1}{m} < \frac{\varepsilon}{2}.$

Por lo anterior, si $n$, $m > k$, entonces

\begin{align*}
\left\lvert \frac{1}{n}-\frac{1}{m} \right\rvert & \leq \frac{1}{n} + \frac{1}{m} \\
& < \frac{\varepsilon}{2} + \frac{\varepsilon}{2} \\
& = \varepsilon.
\end{align*}

$$\therefore \left\lvert \frac{1}{n}-\frac{1}{m} \right\rvert < \varepsilon.$$

Se concluye que $\{\frac{1}{n}\}$ es una sucesión de Cauchy.

$\square$

Ejemplo 2. Prueba que la sucesión $\{ \frac{n}{n+1} \}$ es de Cauchy.

Demostración.

Sea $\varepsilon > 0.$ Consideremos $k \in \mathbb{N}$ tal que $k > \frac{2}{\varepsilon}$, y por tanto $ \frac{1}{k} < \frac{\varepsilon}{2}$. Además, si $n$, $m > k$, entonces se sigue que

\begin{align*}
\left| \frac{n}{n+1} – \frac{m}{m+1} \right| & = \left| \frac{nm+n-nm-m}{(n+1)(m+1)} \right| \\ \\
& = \left| \frac{n-m}{(n+1)(m+1)} \right| \\ \\
& \leq \left| \frac{n-m}{nm} \right| , \text{ pues } (n+1)(m+1) > nm \\ \\
& = \left| \frac{n}{nm} – \frac{m}{nm} \right| \\ \\
& = \left| \frac{1}{m} – \frac{1}{n} \right| \\ \\
& \leq \frac{1}{n} + \frac{1}{m} \\ \\
& < \frac{1}{k} + \frac{1}{k}, \text{ pues } n,m > k \\ \\
& < \frac{\varepsilon}{2} + \frac{\varepsilon}{2}, \text{ pues } \frac{1}{k} < \frac{\varepsilon}{2} \\ \\
& = \varepsilon.
\end{align*}

Por tanto, $\{ \frac{n}{n+1} \}$ es de Cauchy.

$\square$

Ejemplo 3. Demuestra que la sucesión $\{ (-1)^n \}$ no es de Cauchy.

Demostración.

Debemos probar que existe $\varepsilon > 0$ tal que para todo $k \in \mathbb{N}$, existe al menos un $n > k$ y al menos un $m > k$, tales que $|a_n-a_m| \geq \varepsilon$.

Notemos que la sucesión toma el valor $1$ cuando $n$ es par y $-1$ cuando $n$ es impar. Así, consideremos $\varepsilon = 2$ y sea $k \in \mathbb{N}$. Tomemos cualquier número par $n$ tal que $n > k$ y sea $m = n+1$, $m$ impar. Entonces se tiene que

$$|a_n-a_m| = |1-(-1)| = 2 = \varepsilon.$$

Por tanto, se puede concluir que $\{(-1)^n\}$ no es de Cauchy.

$\square$

Proposición. Si $\{a_n\}$ y $\{b_n\}$ son sucesiones de Cauchy, entonces la sucesión $\{a_n+b_n\}$ también es de Cauchy.

Demostración.

Sea $\varepsilon > 0$. Como $\{a_n\}$ es de Cauchy, para $\frac{\varepsilon}{2} > 0$ existe $k_1$ tal que si $n$, $m > k_1$, se tiene que $$|a_n-a_m| < \frac{\varepsilon}{2}.$$

Como $\{b_n\}$ es de Cauchy, para $\frac{\varepsilon}{2} > 0$ existe $k_2$ tal que si $n$, $m > k_2$, se tiene que $$|b_n-b_m| < \frac{\varepsilon}{2}.$$

Consideremos $k = max\{k_1, k_2 \}$, si $n$, $m > k$, entonces

\begin{align*}
|(a_n+b_n)-(a_m+b_m)| & = |(a_n-a_m)+(b_n-b_m)| \\
& \leq |a_n-a_m| + |b_n-b_m| \\
& < \frac{\varepsilon}{2} + \frac{\varepsilon}{2} = \varepsilon.
\end{align*}

$$\therefore |(a_n+b_n)-(a_m+b_m)| < \varepsilon.$$

Por lo tanto, la sucesión $\{a_n+b_n\}$ también es de Cauchy.

$\square$

Una de las propiedades naturales de las sucesiones de Cauchy es que son sucesiones acotadas; esto derivado directamente de la definición donde debe existir un punto $k$ a partir de donde cualesquiera dos términos deben distar menos de $\varepsilon$. A continuación demostraremos tal propiedad.

Proposición. Toda sucesión de Cauchy está acotada.

Demostración.

Sea $\{a_n\}$ una sucesión de Cauchy. Entonces para $\varepsilon = 1$, existe $k \in \mathbb{N}$ tal que para $n \geq k$, se tiene que $|a_n – a_k| < \varepsilon = 1$. De la desigualdad del triángulo se tiene que

\begin{gather*}
& |a_n|-|a_k| \leq |a_n-a_k| < 1. \\
\Rightarrow & |a_n| \leq 1 + |a_k|.
\end{gather*}

Notemos que $1+|a_k|$ es una cota para los términos subsecuentes de $a_k$. Para extender la cota a los primeros $k-1$ términos, consideremos $M = max\{ |a_1|, |a_2|, \ldots, |a_{k-1}|, 1+|a_k|\}$. De esta forma, para todo $n \in \mathbb{N}$ se tiene que $|a_n| < M$. Por tanto, la sucesión está acotada.

$\square$

Es importante resaltar que no es equivalente que una sucesión sea de Cauchy a que cumpla que la distancia entre dos términos consecutivos sea cada vez menor, y lo veremos en el siguiente ejemplo.

Ejemplo 4. Sea $\{a_n\}$ tal que $a_n = \sqrt{n}$. Prueba que la sucesión $\{a_n\}$ satisface que

$$\lim_{n \to \infty} |a_{n+1}-a_n| = 0.$$

Pero que no es una sucesión de Cauchy.

Demostración.

Notemos que

\begin{align*}
|a_{n+1}-a_n| & = \left\lvert \sqrt{n+1}-\sqrt{n} \right\rvert \\ \\
& = \left( \sqrt{n+1}-\sqrt{n} \right) \cdot \frac{\sqrt{n+1}+\sqrt{n}}{\sqrt{n+1}+\sqrt{n}} \\ \\
& = \frac{n+1-n}{\sqrt{n+1}+\sqrt{n}} \\ \\ & = \frac{1}{\sqrt{n+1}+\sqrt{n}}.
\end{align*}

Por lo anterior, se sigue que
$$\lim_{n \to \infty} |a_{n+1}-a_n| = \frac{1}{\sqrt{n+1} + \sqrt{n}} = 0.$$

Por otro lado, se tiene que la sucesión $\{a_n\}$ no está acotada, por lo cual no puede ser una sucesión de Cauchy.

$\square$

Relación entre sucesiones convergentes y de Cauchy

Como se menciona anteriormente, dentro del conjunto de los números reales, que una sucesión sea de Cauchy es equivalente a que sea convergente y a este hecho se le suele llamar Completitud de $\mathbb{R}$. Este tema se estudiará en cursos más avanzados, pero de forma intuitiva, que $\mathbb{R}$ sea completo quiere decir que «no deja huecos en la recta numérica». Es decir, a cada punto de la recta le corresponde un número real.

Teorema. Si $\{a_n\}$ es una sucesión convergente de números reales, entonces es de Cauchy.

Demostración

Sea $\varepsilon > 0$. Dado que $\{a_n\}$ es convergente, digamos a $L$, entonces para $\frac{\varepsilon}{2}$ existe un número $n_0 \in \mathbb{N}$ tal que si $n \geq n_0$ se satisface que $|a_n – L | < \frac{\varepsilon}{2}.$

Consideremos $k = n_0$. Si $n$, $m \geq k$, entonces

\begin{align*}
|a_n-a_m| & = |a_n-L+L-a_m| \\
& \leq|a_n-L| + |L-a_m| \\
& = |a_n-L| + |a_m – L| \\
& < \frac{\varepsilon}{2} + \frac{\varepsilon}{2} \\
& = \varepsilon.
\end{align*}

$$\therefore |a_n-a_m| < \varepsilon.$$

Por lo tanto, $\{a_n\}$ es de Cauchy.

$\square$

Teorema. Toda sucesión de Cauchy es convergente.

Demostración.

Sea $\{a_n\}$ una sucesión de Cauchy. Por la proposición revisada anteriormente, $\{a_n\}$ está acotada. Además, por el teorema de Bolzano-Weierstrass, existe una subsucesión $\{a_{n_r}\}$ convergente y llamemos $L$ al límite de tal subsucesión. Probaremos que $\{a_n\}$ también converge a $L$.

Sea $\varepsilon > 0$. Como la sucesión $\{a_n\}$ es de Cauchy, entonces existe $k \in \mathbb{N}$ tal que
$$|a_n-a_m| < \varepsilon \quad \forall n,m \geq k. \tag{1}$$

Por otro lado, como $\{a_{n_r} \}$ converge a $L$, existe $n_0 \in \mathbb{N}$ tal que

$$|a_{n_l} – L| < \varepsilon \quad \forall n_l \geq n_0. \tag{2}$$

Consideremos $M = max\{k, n_0\}$. Si $s \geq M \geq n_0$, entonces se cumple $(2)$ y además sabemos que $n_s \geq s \geq M \geq k$, pues $n_l$ es una sucesión creciente de números naturales. Por tanto, también se cumple $(1)$. De esto se sigue que

\begin{align*}
|a_s-L|& = |a_s-a_{n_s}+a_{n_s}-L| \\
& \leq |a_s-a_{n_s}|+|a_{n_s}-L| \\
& \leq \frac{\varepsilon}{2} +\frac{\varepsilon}{2} \\
& = \varepsilon.
\end{align*}

$$\therefore |a_s-L| < \varepsilon \quad \forall s \geq M.$$

Se concluye que $\{a_n\}$ es convergente.

$\square$

Más adelante…

Uno de los números más famosos en matemáticas y que probablemente has escuchado hablar de él es el número de Euler: $e$. En la siguiente entrada estudiaremos este número a través de sucesiones y probaremos algunas de sus propiedades.

Tarea moral

A continuación hay algunos ejercicios para que practiques los conceptos vistos en esta entrada. Te será de mucha utilidad intentarlos para entender más la teoría vista.

  • Da un ejemplo de una sucesión de Cauchy.
  • Da un ejemplo de una sucesión acotada que no sea una sucesión de Cauchy.
  • Prueba mediante la definición que la sucesión $\{ \frac{n+1}{n} \}$ es de Cauchy.
  • Demuestra mediante la definición que la sucesión $\{ (-1)^n \}$ no es de Cauchy.
  • Demuestra mediante la definición que si $\{a_n\}$ y $\{b_n\}$ son sucesiones de Cauchy, entonces la sucesión $\{a_n \cdot b_n\}$ también es de Cauchy.

Entradas relacionadas

Agradecimientos

Trabajo realizado con el apoyo del Programa UNAM-DGAPA-PAPIME PE104522 «Hacia una modalidad a distancia de la Licenciatura en Matemáticas de la FC-UNAM – Etapa 2»

Cálculo Diferencial e Integral I: Subsucesiones

Por Juan Manuel Naranjo Jurado

Introducción

Dada una sucesión, si «quitamos» cierta cantidad de términos de tal forma que aún queda una cantidad infinita de ellos y se conserva el orden de la sucesión original, se genera un tipo particular de sucesión llamado subsucesión. En esta entrada probaremos algunas de sus características y veremos cómo se enlazan sus propiedades respecto a la sucesión original.

Subsucesiones

Primero formalizaremos la idea intuitiva dada en la introducción a través de la siguiente definición.

Definición. Sea $\{a_n\}$ una sucesión de números reales y sea $n_1 < n_2 < \ldots < n_k < \ldots$, con $k \in \mathbb{N}$, una sucesión estrictamente creciente de números naturales. Entonces la sucesión $\{ a_{n_k} \}$ dada por $$\{ a_{n_1}, a_{n_2}, \ldots, a_{n_k}, \ldots \}$$

es una subsucesión de $\{a_n\}$.

Observación. Es importante recalcar que en la definición se indica que los índices de los términos de la subsucesión son una sucesión por sí mismos. Esto se podrá apreciar claramente en los ejemplos siguientes.

Ejemplo 1. Consideremos la sucesión $\{ a_n \} = \{ \frac{1}{n} \}$.

Si tomamos los términos con índice par, obtenemos la subsucesión $\{ a_2, a_4, a_6, \ldots, a_{2k}, \ldots \}$ cuyos términos son

$$\left\lbrace \frac{1}{2}, \frac{1}{4}, \frac{1}{6}, \ldots, \frac{1}{2k}, \ldots \right\rbrace.$$

De esta forma, se tiene que $n_1 = 2$, $n_2 = 4$, $\ldots$, $n_k = 2k$, $\ldots$. Y podemos observar que los índices forman una sucesión estrictamente creciente, es decir, se cumple que $$n_1 < n_2 < \ldots < n_k < \ldots.$$

Si consideramos ahora $n_k = 2k-1$, obtenemos una nueva subsucesión $\{ a_1, a_3, a_5, \ldots, a_{2k-1}, \ldots \}$ conformada por los términos

$$\left\lbrace \frac{1}{1}, \frac{1}{3}, \frac{1}{5}, \ldots, \frac{1}{2k-1}, \ldots \right\rbrace.$$

Otra subsucesión podría ser la generada por los índices $n_k = k^2$, generando la subsucesión $\{ a_1, a_4, a_9, \ldots, a_{k^2} \}$, con términos

$$\left\lbrace \frac{1}{1}, \frac{1}{4}, \frac{1}{9}, \ldots, \frac{1}{k^2}, \ldots \right\rbrace.$$

En contraste, podemos observar que $\{a_2, a_1, a_4, a_3, \ldots, a_{k+1}, a_{k-1}, \ldots \}$ no es una subsucesión de $\{ a_n \}$. Los términos generados son

$$\left\lbrace \frac{1}{2}, \frac{1}{1}, \frac{1}{4}, \frac{1}{3}, \ldots, \frac{1}{k+1}, \frac{1}{k-1}, \ldots \right\rbrace.$$

Y no es subsucesión debido a que ésta no respeta el orden de la sucesión original, en otras palabras, la sucesión de índices $\{n_k\}$ no es estrictamente creciente. En este caso
$$n_k = \begin{cases} k+1 & \text{ si } k \text{ es impar} \\ k-1 & \text{ si } k \text{ es par}. \end{cases}$$

Con lo cual podemos ver que $n_1 = 2$, $n_2=1$, $n_3 = 4$, $n_4 = 3$, $\ldots$, por lo que $\{n_k\}$ no es monótona y, particularmente, no es estrictamente creciente.

Una forma singular de crear subsucesiones a partir de una sucesión dada, es eliminando los primeros $m$ términos de la sucesión. Así, tenemos la siguiente definición.

Definición. Sea $\{a_n\}$ una sucesión de números reales y sea $m \in \mathbb{N}$. Definimos la cola-$m$ de $\{a_n\}$ como la sucesión

$$\{a_{m+n}: n \in \mathbb{N}\} = \{ a_{m+1}, a_{m+2}, \ldots \}.$$

La cola-$m$ es una subsucesión donde $n_1 = m+1$, $n_2 = m +2$, $\ldots$, $n_k = m+k$.

Ejemplo 2. Consideremos la sucesión $\{a_n\} = \{ \sqrt{n} \}.$

La cola-$10$ de $\{a_n\}$ es la subsucesión $\{ a_{11}, a_{12}, \ldots, a_k \ldots \}$, cuyos términos son $\{ \sqrt{11}, \sqrt{12}, \ldots, \sqrt{k}, \ldots \}.$

Subsucesiones de sucesiones convergentes

Si generamos una subsucesión de una sucesión convergente, es natural que dicha subsucesión también converja y, de hecho, lo hace al límite de la sucesión original.

Teorema. Si una sucesión $\{a_n\}$ de números reales converge a un número real $L$, entonces cualquier subsucesión $\{ a_{n_k}\}$ también converge a $L$.

Demostración.

Sea $\varepsilon > 0$. Como $\{a_n\}$ converge a $L$, entonces existe un número natural $n_0 \in \mathbb{N}$ tal que para todo $n \geq n_0$ se tiene que

$$|a_n – L| < \varepsilon.$$

Dado que $n_1 < n_2 < \ldots < n_k < \ldots$ es una sucesión creciente de números naturales, se tiene que $n_k \geq k$. De esta forma, tenemos que si $k \geq n_0$, entonces $n_k \geq k \geq n_0$. Por lo que se cumple que
$$|a_{n_k} – L| < \varepsilon.$$

Por lo tanto, la sucesión $\{ a_{n_k} \}$ también converge a $L$.

$\square$

Ejemplo 3. Del teorema anterior se sigue que dada una sucesión $\{a_n\}$ convergente a $L$, la cola-$m$ de la sucesión también converge a $L$ para todo $m \in \mathbb{N}$.

Ejemplo 4. Consideremos la sucesión $\{a_n\} = \{ \frac{1}{\pi^n}\}$.

Notemos que $\frac{1}{\pi^n} = \left( \frac{1}{\pi} \right)^n$.

Además, como $\pi > 1$, entonces $\frac{1}{\pi} <1$. Por tanto $$\lim_{n \to \infty} \frac{1}{\pi^n} = 0.$$

Así, toda subsucesión de $\{a_n\}$ converge a cero. Podemos considerar, por ejemplo, la subsucesión generada tomando $n_k = 2k$, es decir, la subsucesión $\{a_{n_k} \} = \{ \frac{1}{\pi^{2k}} \}$ converge a cero.

Subsucesiones y la no convergencia

Hasta este punto hemos revisado las subsucesiones y su relación con la convergencia; ahora es momento de encontrar qué sucede respecto a la no convergencia.

Teorema. Sea $\{a_n\}$ una sucesión de números reales. Entonces los siguientes enunciados son equivalentes:

  1. La sucesión $\{a_n\}$ no converge a $L \in \mathbb{R}.$
  2. Existe $\varepsilon_0 > 0$ tal que para cualquier $k \in \mathbb{N}$, existe $n_k \in \mathbb{N}$ tal que $n_k \geq k$ y $|a_{n_k} – L| \geq \varepsilon_0.$
  3. Existe $\varepsilon_0 > 0$ y una subsucesión $\{a_{n_k}\}$ de $\{a_n\}$ tal que $|a_{n_k} – L| \geq \varepsilon_0$ para todo $k \in \mathbb{N}.$

Demostración.

$1 \Rightarrow 2]$ Si $\{a_n\}$ no converge, entonces existe $\varepsilon_0 > 0$ para el cual no es posible encontrar un natural $k$ tal que para todo $n \geq k$ se cumpla $|a_n-L| < \varepsilon$. Es decir, para todo $k \in \mathbb{N}$ existe un natural $n_k \geq k$ tal que $|a_{n_k} – L | \geq \varepsilon_0.$

$2 \Rightarrow 3]$ Sea $\varepsilon_0 > 0$ tal que cumple $2)$ y sea $n_1 \in \mathbb{N}$ tal que $n_1 \geq 1$ y $|a_{n_1} – L| \geq \varepsilon_0.$

Ahora sea $n_2 \in \mathbb{N}$ tal que $n_2 > n_1$ y $|a_{n_2} – L| \geq \varepsilon_0.$

Sea $n_3 \in \mathbb{N}$ tal que $n_3 > n_2$ y $|a_{n_3} – L| \geq \varepsilon_0.$

Se continúa de esta manera para obtener la subsucesión $\{a_{n_k}\}$ tal que $|a_{n_k} – L| \geq \varepsilon_0$ para todo $k \in \mathbb{N}.$

$3 \Rightarrow 1]$ Supongamos que $\{a_n\}$ tiene una subsucesión $\{a_{n_k}\}$ que satisface la condición $3)$. Entonces $\{a_n\}$ no puede converger a $L$ porque sería una contradicción al teorema anterior.

$\square$

Criterios de no convergencia. Sea $\{a_n\}$ una sucesión de números reales. Si se cumple cualquiera de las siguientes condiciones, entonces la sucesión es divergente.

  1. $\{a_n\}$ tiene dos subsucesiones convergentes $\{a_{n_k}\}$ y $\{a_{n_l}\}$. Donde $\{a_{n_k}\}$ converge $L$ y $\{a_{n_l}\}$ converge a $M$, pero $L \neq M.$
  2. $\{a_n\}$ no está acotada.

Ejemplo 5. Prueba que la sucesión $\{(-1)^n\}$ no es convergente.

Demostración.

Consideremos las subsucesiones $\{(-1)^{2k}\}$ y $\{(-1)^{2k-1}\}$. Es claro que la primera subsucesión converge a $1$, mientras que la segunda converge a $-1$. Por tanto, la sucesión no converge.

$\square$

Ejemplo 6. Prueba que la sucesión $\{n!\}$ no es convergente.

Demostración.

Dado que $n! \geq n$ para todo $n \in \mathbb{N}$, y sabemos que la sucesión generada por los números naturales no está acotada. Se sigue que la sucesión $\{n!\}$ no está acotada. Por tanto, no es convergente.

$\square$

Ejemplo 7. Prueba que la sucesión $\{ 1 – (-1)^n + \frac{1}{n} \}$ no es convergente.

Demostración.

Consideremos las subsucesiones $\{ 1 – (-1)^{2k} + \frac{1}{2k} \}$ y $\{ 1 – (-1)^{2k-1} + \frac{1}{2k-1} \}.$

Notemos que

\begin{align*}
\lim_{k \to \infty } 1 – (-1)^{2k} + \frac{1}{2k} & = \lim_{k \to \infty } 1 – ((-1)^2)^k + \frac{1}{2k} \\ \\
& = \lim_{k \to \infty } 1 – (1)^k + \frac{1}{2k} \\ \\
& = 1-1+0 \\ \\
& = 0.
\end{align*}

Análogamente, se tiene que

\begin{align*}
\lim_{k \to \infty } 1 – (-1)^{2k-1} + \frac{1}{2k-1} & = \lim_{k \to \infty } 1 – (-1)^{2k} \cdot (-1)^{-1} + \frac{1}{2k-1} \\ \\
& = \lim_{k \to \infty } 1 – (1)^k \cdot (-1) + \frac{1}{2k-1} \\ \\
& = 1+1+0 \\ \\
& = 2.
\end{align*}

Por tanto, la primera subsucesión converge a $0$, mientras que la segunda converge a $2$. Se concluye que la sucesión $\{ 1 – (-1)^n + \frac{1}{n} \}$ no es convergente.

$\square$

Teorema de Bolzano-Weierstrass

El teorema de Bolzano-Weierstrass nos indica que toda sucesión acotada tiene una subsucesión convergente. Un ejemplo claro es el revisado en esta entrada, la sucesión $\{(-1)^n\}$ de la cual hemos probado anteriormente que está acotada y es fácil notar que la subsucesión generada por los índices pares $n_k =2k$ es convergente. Sin embargo, para probar el caso general, veremos primero que toda sucesión tiene una subsucesión monótona y, usando un teorema previamente revisado que indica que toda sucesión monótona acotada es convergente, podremos probar fácilmente el teorema de Bolzano-Weierstrass.

Teorema. Si $\{a_n\}$ es una sucesión de números reales, entonces existe una subsucesión $\{a_{n_k} \}$ que es monótona.

Demostración.

Por practicidad, diremos que $a_m$ es un «pico» si $a_m \geq a_n$ para todo $n \geq m$. Es decir, $a_m$ nunca es excedido por ningún término posterior en la sucesión. Tal como se muestra en la siguiente ilustración, donde los puntos rojos representan los «picos» de la sucesión.

Podemos notar que en una sucesión decreciente cualquier término es un pico, mientras que para una sucesión creciente ningún término es un pico. Dada una sucesión, podemos dividir en dos casos de acuerdo a la cantidad de picos que ésta posea.

  • Caso 1: La sucesión tiene una cantidad infinita de picos.
    En este caso, la enumeración de los picos se hace con subíndices crecientes: $a_{m_1}$, $a_{m_2}$, $\ldots$, $a_{m_k}$, $\ldots$ Puesto que cada término es un pico se tiene que
    $$ a_{m_1} \geq a_{m_2} \geq \ldots \geq a_{m_k}.$$
    Por tanto, la subsucesión $\{ a_{m_k}\}$ es una subsucesión decreciente de $\{ a_n\}$.

  • Caso 2: La sucesión tiene una cantidad finita de picos.
    Nuevamente, la sucesión generada por los subíndices es creciente: $a_{m_1}$, $a_{m_2}$, $\ldots$, $a_{m_k}$. Sea $s_1 = m_k+1$ el primer índice después del último pico, entonces existe $s_2 > s_1$ tal que $a_{s_1} < a_{s_2}$ dado que $a_{s_1}$ no es un pico. Además, sucede que $a_{s_2}$ tampoco es un pico, por lo que existe $s_3 > s_2$ tal que $a_{s_2} < a_{s_3}$. Al continuar de esta forma, se obtiene una subsucesión creciente $\{ a_{s_k}\}$ de $\{a_n\}.$

De ambos casos podemos concluir que toda sucesión tiene una subsucesión monótona.

$\square$

Teorema de Bolzano-Weierstrass. Toda sucesión acotada de números reales tiene una subsucesión convergente.

Demostración.

Sea $\{a_n\}$ una sucesión acotada. Por el teorema anterior, sabemos que $\{a_n\}$ tiene una subsucesión monótona $\{a_{n_k}\}$, además la subsucesión también está acotada pues $\{a_n\}$ lo está, entonces $\{a_{n_k}\}$ es convergente.

$\square$

Como último teorema, revisaremos que si toda subsucesión convergente de una sucesión acotada tiene límite $L$, entonces debe suceder que la sucesión original también converja a $L$.

Teorema. Sea $\{a_n\}$ una sucesión acotada de números reales. Si toda subsucesión convergente $\{a_{n_k} \}$ de $\{a_n\}$ converge a $L$, entonces $\{a_n\}$ también converge y lo hace a $L$.

Demostración.

Sea $\{a_n\}$ una sucesión acotada tal que todas sus subsucesiones convergentes lo hacen a $L$. Entonces existe $M \in \mathbb{R}$ tal que $|a_n| < M$ para todo $n \in \mathbb{N}$.

Supongamos que $\{a_n\}$ no converge a $L$, entonces existen $\varepsilon_0 > 0$ y una subsucesión $\{a_{n_k}\}$ tal que

$$|a_{n_k} – L| \geq \varepsilon_0 \quad \forall k \in \mathbb{N}. \tag{1}$$

Puesto que $\{ a_{n_k}\}$ es una subsucesión de $\{a_n\}$, entonces también se cumple que $|a_{n_k}| < M $. Es decir, $M$ también es una cota $\{ a_{n_k}\}$. Por el teorema de Bolzano-Weierstrass, esto implica que $\{a_{n_k}\}$ tiene una subsucesión convergente $\{a_{r_k}\}$. Puesto que esta última subsucesión también es subsucesión de $\{a_n\}$ converge a $L$ por hipótesis. Por tanto, existe $n_0 \in \mathbb{N}$ tal que para $r_k \geq n_0$ se tiene que $|a_{r_k} – L| \leq \varepsilon_0$, lo cual contradice $(1).$

$\square$

Más adelante…

En la siguiente entrada estudiaremos las sucesiones de Cauchy, las cuales nos permitirán dar un enfoque especial a las sucesiones convergentes donde no será necesario conocer a priori el valor del límite. Además, probaremos la equivalencia existente entre las sucesiones convergentes y las sucesiones de Cauchy.

Tarea moral

A continuación hay algunos ejercicios para que practiques los conceptos vistos en esta entrada. Te será de mucha utilidad intentarlos para entender más la teoría vista.

  • Da un ejemplo de una sucesión y dos subsucesiones de ella.
  • Da un ejemplo de una sucesión acotada que tenga una subsucesión convergente.
  • Da un ejemplo de una sucesión no acotada que tenga una subsucesión convergente.
  • Prueba que la sucesión $\{ \left( 1 + \frac{1}{n^2} \right) ^{n^2} \}$ es convergente.
  • Determina el límite de la sucesión$\{ (3n)^{\frac{1}{2n}} \}$.

Entradas relacionadas

Agradecimientos

Trabajo realizado con el apoyo del Programa UNAM-DGAPA-PAPIME PE104522 «Hacia una modalidad a distancia de la Licenciatura en Matemáticas de la FC-UNAM – Etapa 2»

Cálculo Diferencial e Integral I: Sucesiones monótonas

Por Juan Manuel Naranjo Jurado

Introducción

En esta entrada terminaremos de revisar las operaciones aritméticas de sucesiones probando qué sucede con el cociente de sucesiones convergentes. Además, daremos la definición de sucesión monótona y demostraremos algunas de sus propiedades.

Cociente de sucesiones

Daremos inicio demostrando que el cociente de sucesiones convergentes converge al cociente de sus límites siempre que se cumpla la condición de que el denominador sea una sucesión de términos distintos de cero e, igualmente, debe tener como límite a un número real distinto de cero.

Proposición. Sean $\{b_n \}$ una sucesión en los reales tal que
$$\lim_{n \to \infty} b_n = M.$$
Si además se tiene que $M \neq 0$ y $b_n \neq 0$ para todo $n \in \mathbb{N}$, entonces
$$\lim_{n \to \infty} \frac{1}{b_n} = \frac{1}{M}.$$

Demostración.

Sea $\varepsilon > 0$. Para $n \in \mathbb{N}$, se tiene

\begin{align*}
\left\lvert \frac{1}{b_n} – \frac{1}{M} \right\rvert &
= \left\lvert \frac{M-b_n}{M \cdot b_n} \right\rvert \\ \\
& = \frac{|M-b_n|}{|M \cdot b_n|} \\ \\
& = \frac{1}{|M \cdot b_n|} \cdot |M-b_n| \\ \\
& = \frac{1}{|M|} \cdot \frac{1}{| b_n|} \cdot |M-b_n|.
\end{align*}

$$\therefore \left\lvert \frac{1}{b_n} – \frac{1}{M} \right\rvert = \frac{1}{|M|} \cdot \frac{1}{|b_n|} \cdot |M-b_n|. \tag{1}$$

Sea $\frac{|M|}{2} > 0$, como $\{b_n\}$ converge a $M$, entonces existe $n_1 \in \mathbb{N}$ tal que si $n \geq n_1$ se cumple que
\begin{gather*}
|M-b_n| < \frac{|M|}{2}.
\end{gather*}
Se sigue que
$$ |M| – |b_n | \leq |M-b_n| < \frac{|M|}{2}. $$
De la expresión anterior se obtiene que $$\frac{|M|}{2} < |b_n|.$$
Por tanto,
$$\frac{1}{|b_n|} < \frac{2}{|M|}. \tag{2}$$

De $(1)$ y $(2)$ se tiene que si $n \geq n_1$, entonces

\begin{align*}
\left\lvert \frac{1}{b_n} – \frac{1}{M} \right\rvert & = \frac{1}{|M|} \cdot \frac{1}{| b_n|} \cdot |M-b_n| \\ \\
& < \frac{1}{|M|} \cdot \frac{2}{|M|} \cdot |M-b_n| \\ \\
& = \frac{2}{|M|^2} \cdot |M-b_n|.
\end{align*}

$$\therefore \left\lvert \frac{1}{b_n} – \frac{1}{M} \right\rvert < \frac{2}{|M|^2} \cdot |M-b_n|. \tag{3}$$

Ahora consideremos $$\frac{\varepsilon}{\frac{2}{|M|^2}} > 0.$$

Nuevamente, como $\{ b_n \}$ converge a $M$, existe $n_2 \in \mathbb{N}$ tal que si $n \geq n_2$, entonces $$|b_n-M| < \frac{\varepsilon}{\frac{2}{|M|^2}}. \tag{4}$$

Tomemos $n_0 = max\{n_1, n_2 \}$. Si $n \geq n_0$, también se cumple que $n \geq n_1$ y $n \geq n_2$ y de $(3)$ y $(4)$ se tiene que
\begin{align*}
\left\lvert \frac{1}{b_n} – \frac{1}{M} \right\rvert & < \frac{2}{|M|^2} \cdot |M-b_n| \\ \\
& = \frac{2}{|M|^2} \cdot |b_n-M| \\ \\
&< \frac{2}{|M|^2} \cdot \frac{\varepsilon}{\frac{2}{|M|^2}} \\ \\
& = \varepsilon.
\end{align*}

$$ \therefore \left\lvert \frac{1}{b_n} – \frac{1}{M} \right\rvert < \varepsilon.$$

$$\therefore \lim_{n \to \infty} \frac{1}{b_n} = \frac{1}{M}.$$

$\square$

Ejemplo 1. Prueba el siguiente límite $$\lim_{n \to \infty} \frac{3}{1+(n+10)^2}-1 = -1.$$

Demostración.

Primero desarrollaremos la expresión y multiplicaremos por un uno, $\frac{ \frac{1}{n^2} }{ \frac{1}{n^2} }$, que nos ayudará a obtener límites que ya conocemos. Podemos observar que el uno que usaremos está bien definido dado que $n \in \mathbb{N}$, por lo que $n \geq 1$.

\begin{align*}
\frac{3}{1+(n+10)^2}-1 & = \frac{3}{n^2+20n+101}-1 \\ \\
& = \frac{3}{n^2+20n+101} \cdot \frac{ \frac{1}{n^2} }{ \frac{1}{n^2} } -1 \\ \\
& = \frac{\frac{3}{n^2}}{\frac{n^2+20n+101}{n^2}}-1 \\ \\
& = \frac{\frac{3}{n^2}}{1+\frac{20}{n}+\frac{101}{n^2}}-1. \\ \\
\end{align*}

Además, sabemos que $$ \lim_{n \to \infty} \frac{1}{n} = 0.$$

Y usando las propiedades que hemos visto respecto a las operaciones de sucesiones convergentes, se tiene que
\begin{align*}
\lim_{n \to \infty} \frac{3}{1+(n+10)^2}-1 & = \lim_{n \to \infty} \frac{\frac{3}{n^2}}{1+\frac{20}{n}+\frac{101}{n^2}}-1 \\ \\
&= \frac{\lim_{n \to \infty} \frac{3}{n^2}}{\lim_{n \to \infty} 1+\frac{20}{n}+\frac{101}{n^2} } – \lim_{n \to \infty} 1 \\ \\
& = \frac{0}{1+0+0}-1 \\ \\
& = -1.
\end{align*}

$$\therefore \lim_{n \to \infty} \frac{3}{1+(n+10)^2}-1 = -1.$$

$\square$

Sucesiones monótonas

A continuación daremos algunas definiciones referentes a la monotonía que se presenta en las sucesiones.

Definición. Sea $\{a_n \}$ una sucesión de números reales.

  • Se dice que la sucesión es creciente si satisface que $a_n \leq a_{n+1}$ para todo $n \in \mathbb{N}$. Si la desigualdad es estricta, se dice que la sucesión es estrictamente creciente.
  • Se dice que la sucesión es decreciente si satisface que $a_n \geq a_{n+1}$ para todo $n \in \mathbb{N}$. Si la desigualdad es estricta, se dice que la sucesión es estrictamente decreciente.
  • Se dice que la sucesión es monótona si es creciente o decreciente. Si la desigualdad es estricta, se dice que la sucesión es estrictamente monótona.

Ejemplo 2. Las siguientes sucesiones son decrecientes:

  • $\{\frac{1}{n}\}.$
  • $\{\frac{1}{n!}\}.$
  • $\{c^n\}$ si $0< c < 1.$
  • $\{\frac{1}{2^n}\}.$

Probaremos la monotonía de la última sucesión.

Demostración.

Sea $\{a_n\} = \{\frac{1}{2^n}\}$. Consideremos $n \in \mathbb{N}$, sabemos que $2 > 1$ y $2^n > 0$, entonces se tiene que

\begin{gather*}
& 2 \cdot 2^n > 1 \cdot 2^n. \\ \\
\Leftrightarrow & 2^{n+1} > 2^n. \\ \\
\Leftrightarrow & \frac{1}{2^n} > \frac{1}{2^{n+1}}.
\end{gather*}

$$\therefore a_n \geq a_{n+1}.$$

Además, como se cumple la desigualdad estricta, la sucesión $\{a_n\} = \{\frac{1}{2^n}\}$ es estrictamente decreciente.

$\square$

Ejemplo 3. Las siguientes sucesiones son crecientes:

  • $\{n\}.$
  • $\{n^2\}.$
  • $\{c^n\}$ si $c > 1.$
  • $\{ \sqrt{n} \}.$

Probaremos la monotonía de la última sucesión.

Demostración.

Sea $\{a_n\} = \{ \sqrt{n} \}$. Consideremos $n \in \mathbb{N}$, sabemos que $n <n+ 1$, entonces se tiene que

\begin{gather*}
& 0 < n+1-n. \\
\Leftrightarrow & 0 < \left( \sqrt{n+1} \right)^2 – \left( \sqrt{n} \right)^2. \\ \\
\Leftrightarrow & 0 < \left( \sqrt{n+1} – \sqrt{n}\right) \left( \sqrt{n+1} + \sqrt{n}\right). \\ \\
\Leftrightarrow & \frac{0}{ \sqrt{n+1} + \sqrt{n} } < \sqrt{n+1} – \sqrt{n}, \text{pues } \sqrt{n+1} + \sqrt{n} > 0.
\end{gather*}

De la expresión anterior se sigue que
$$0 < \sqrt{n+1} – \sqrt{n}. $$
Es decir,
$$\sqrt{n} < \sqrt{n+1}.$$

$$\therefore a_{n} \leq a_{n+1}.$$

Además, como se cumple la desigualdad estricta, la sucesión $\{a_n\} = \{ \sqrt{n} \}$ es estrictamente creciente.

$\square$

Una vez dada la definición, podemos probar el siguiente teorema.

Teorema. Una sucesión monótona de números reales es convergente si y solo si está acotada. Además,

  1. Si $\{a_n\}$ es una sucesión creciente acotada, entonces
    $$ \lim_{n \to \infty} a_n = sup\{a_n : n \in \mathbb{N} \}.$$
  2. Si $\{a_n\}$ es una sucesión decreciente acotada, entonces
    $$ \lim_{n \to \infty} a_n = inf\{a_n : n \in \mathbb{N} \}.$$

Demostración.

$\Rightarrow]$ En la entrada anterior se probó que toda sucesión convergente está acotada, particularmente una sucesión convergente monótona también está acotada.

$\Leftarrow]$ Sea $\{a_n\}$ una sucesión monótona acotada. Entonces la sucesión es creciente o decreciente.

  • Caso 1: $\{a_n\}$ es creciente.
    Como $\{a_n\}$ está acotada, entonces existe un número real $M$ tal que $a_n \leq M$ para todo $n \in \mathbb{N}$. Sea $A = \{a_n | n \in \mathbb{N} \}$, como $A \neq \varnothing$ y está acotado, entonces existe el supremo. Definimos $\alpha = supA$.

    Sea $\varepsilon > 0$. Notemos que $\alpha – \varepsilon < \alpha$ y como $\alpha$ es la cota superior más pequeña del conjunto $A$, entonces $\alpha – \varepsilon$ no es cota superior de $A$. Entonces existe $n_0 \in \mathbb{N}$ tal que $$\alpha – \varepsilon < a_{n_0} \leq \alpha < \alpha + \varepsilon.$$

    Si $n \geq n_0$, como $\{a_n\}$ es creciente, se tiene que
    \begin{gather*}
    & \alpha – \varepsilon < a_{n_0} \leq a_n \leq \alpha < \alpha + \varepsilon. \\
    \Rightarrow & \alpha – \varepsilon < a_n < \alpha + \varepsilon.
    \end{gather*}

    Es decir, $$-\varepsilon < a_n – \alpha < \varepsilon.$$

    $$\therefore |a_n – \alpha| < \varepsilon \quad \forall n \geq n_0.$$
    $$\therefore \lim_{n \to \infty} a_n = \alpha.$$
  • Caso 2: $\{a_n\}$ es decreciente.
    Quedará como tarea moral.

$\square$

Gracias al teorema anterior, dada una sucesión que sea monótona, basta probar que está acotada para saber que es convergente. Más aún, si determinamos el ínfimo/supremo de tal sucesión, estaremos encontrando su límite; el siguiente ejemplo nos permitirá poner esto en práctica.

Ejemplo 4. Determina el siguiente límite $$\lim_{n \to \infty} \frac{1}{\sqrt{n}}.$$

Demostración.

Sabemos que $\{ \sqrt{n} \}$ es creciente, entonces para todo $n \in \mathbb{N}$
$$\sqrt{n} \leq \sqrt{n+1}.$$

$$ \Leftrightarrow \frac{1}{\sqrt{n+1}} \leq \frac{1}{\sqrt{n}}.$$

Por lo tanto, se tiene que la sucesión $\{ \frac{1}{\sqrt{n}} \}$ es decreciente y tiene como ínfimo el $0$, por tanto, se tiene que

$$\lim_{n \to \infty} \frac{1}{\sqrt{n}} = 0.$$

$\square$

Más adelante…

En la siguiente entrada añadiremos a nuestro arsenal más propiedades de las sucesiones convergentes con lo cual tendremos un estudio más detallado de las mismas.

Tarea moral

A continuación hay algunos ejercicios para que practiques los conceptos vistos en esta entrada. Te será de mucha utilidad intentarlos para entender más la teoría vista.

  • Sean $\{a_n\}$, $\{b_n \}$ dos sucesiones de números reales tal que $$\lim_{n \to \infty} a_n = L \text{ y } \lim_{n \to \infty} b_n = M.$$
    Si además se tiene que $L \neq 0$ y $b_n \neq 0$ para todo $n \in \mathbb{R}$, entonces $$\lim_{n \to \infty} \frac{a_n}{b_n} = \frac{L}{M}.$$
  • Prueba que si $\{a_n\}$ es una sucesión decreciente acotada, entonces
    $$ \lim_{n \to \infty} a_n = inf\{b_n : n \in \mathbb{N} \}.$$
  • Da un ejemplo de sucesión convergente que no sea monótona.
  • Da un ejemplo de sucesión de números reales negativos tal que converja a cero, pero que no sea creciente.
  • Da un ejemplo de sucesión creciente y acotada, y encuentra su límite.

Entradas relacionadas

Agradecimientos

Trabajo realizado con el apoyo del Programa UNAM-DGAPA-PAPIME PE104522 «Hacia una modalidad a distancia de la Licenciatura en Matemáticas de la FC-UNAM – Etapa 2»

Cálculo Diferencial e Integral I: Operaciones con sucesiones convergentes

Por Juan Manuel Naranjo Jurado

Introducción

Después de haber revisado la definición de sucesión convergente y haber calculado el límite de varias sucesiones, es momento de probar algunos resultados que nos permitan conocer qué sucede con las operaciones entre sucesiones convergentes. En esta entrada, demostraremos que la suma y el producto de sucesiones convergentes dan lugar a nuevas sucesiones que convergen a la suma y el producto de los límites respectivamente.

Suma de sucesiones convergentes

Daremos inicio con la prueba de que la suma sucesiones convergentes también converge y lo hace a la suma de los límites correspondientes.

Proposición. Sean $\{a_n \}$ y $\{ b_n \}$ dos sucesiones tales que
$$\lim_{n \to \infty} a_n = L \text{ y } \lim_{n \to \infty} b_n = M.$$
Entonces se tiene
$$\lim_{n \to \infty} (a_n + b_n) = L + M.$$

Demostración.

Empleando la definición de sucesión convergente, lo que debemos demostrar es que para todo $\varepsilon > 0$, existe $n_0 \in \mathbb{N}$ tal que para todo $n \geq n_0$ se satisface que $|(a_n + b_n) – (L + M) |< \varepsilon$.

Iniciamos la prueba dando un valor arbitrario para épsilon. Sea $\varepsilon > 0$.

Notemos que es posible agrupar los términos de tal forma que la expresión quede acotada por $L$ y $M$, como se muestra a continuación:

$$|(a_n + b_n) – (L + M) | = |(a_n – L ) + (b_n – M) | \leq |a_n – L| + |b_n – M|. \tag{1}$$

Además, por hipótesis, ambas sucesiones son convergentes, es decir,
$$\lim_{n \to \infty} a_n = L \text{ y } \lim_{n \to \infty} b_n = M.$$

Por tanto, podemos tomar un valor arbitrario positivo, en este caso consideremos $\frac{\varepsilon}{2} > 0$, de tal forma que para este valor existe $n_1 \in \mathbb{N}$ tal que para todo $n \geq n_1$, se cumple $$|a_n – L | < \frac{\varepsilon}{2}. \tag{2}$$ 

Análogamente, existe $n_2 \in \mathbb{N}$ tal que para todo $n \geq n_2$, se cumple $$|b_n – M | < \frac{\varepsilon}{2}. \tag{3}$$

Consideremos $n_0 = max\{n_1, n_2 \}$. De esta forma, si $n \geq n_0$, se tiene que $n \geq n_1$ y $n \geq n_2$. Por tanto, si $n \geq n_0$, también se cumplen (2) y (3). Entonces

\begin{align*}
|(a_n + b_n) – (L+M)|& \leq |a_n – L| + |b_n – M| \text{, por (1)} \\
& < \frac{\varepsilon}{2} + \frac{\varepsilon}{2} \text{, por (2) y (3)} \\
& = \varepsilon.
\end{align*}

$$\therefore |(a_n + b_n) – (L + M) | < \varepsilon.$$

$$\therefore \lim_{n \to \infty} (a_n + b_n) = L + M.$$

$\square$

A continuación probaremos qué sucede cuando se multiplica una sucesión por un valor real constante.

Proposición. Sea $\{ a_n \}$ una sucesión en $\mathbb{R}$ y $k \in \mathbb{R}$, $k$ fijo, entonces 

$$\lim_{n \to \infty} k \cdot a_n = k \cdot L.$$

Demostración.

Sea $\varepsilon > 0$. Notemos 

$$ |k \cdot a_n – k \cdot L| = |k(a_n – L)| = |k||a_n – L |.$$

Para el caso de $k=0$, la sucesión $\{k \cdot a_n\}$ se convierte en una sucesión constante donde todos sus términos son cero. Así, consideremos $k \neq 0$ y $\frac{\varepsilon}{|k|} > 0 $. Como $$\lim_{n \to \infty} a_n = L,$$ existe $n_0 \in \mathbb{N}$ tal que para todo $n \geq n_0$ se tiene que

\begin{gather*}
& |a_n – L | < \frac{\varepsilon}{|k|}. \\ \\
\Leftrightarrow & |k||a_n – L| < \varepsilon. \\ \\
\therefore & |k \cdot a_n – k \cdot L| = |k||a_n – L | < \varepsilon.
\end{gather*}

$$\therefore |k \cdot a_n – k \cdot L| < \varepsilon.$$

$$\lim_{n \to \infty} k \cdot a_n = k \cdot L.$$

$\square$

Haciendo uso de las dos proposiciones anteriores, podemos probar fácilmente que la diferencia de dos sucesiones convergentes converge a la diferencia de sus límites.

Corolario. Sean $\{a_n \}$ y $\{ b_n \}$ dos sucesiones tales que
$$\lim_{n \to \infty} a_n = L \text{ y } \lim_{n \to \infty} b_n = M.$$
Entonces se tiene
$$\lim_{n \to \infty} (a_n – b_n) = L – M.$$

Demostración.

\begin{align*}
\lim_{n \to \infty} (a_n – b_n) & = \lim_{n \to \infty} (a_n +(- 1 \cdot b_n)) \\
& = \lim_{n \to \infty} a_n +\lim_{n \to \infty} (- 1 \cdot b_n) \\
& = L + (-1 \cdot M) \\
& = L – M.
\end{align*}

$$\therefore \lim_{n \to \infty} (a_n – b_n) = L – M.$$

$\square$

Sucesiones acotadas

Antes de revisar qué sucede con el producto de sucesiones, estudiaremos el concepto de sucesión acotada, lo que significa que existe un número real $M > 0$ que «encierra» a la sucesión. Es decir, existe un intervalo en el cual todos los términos de la sucesión quedan contenidos. De forma ilustrativa, se muestra la gráfica de la sucesión $\left\lbrace \frac{3}{1+(n-10)^2}-1 \right\rbrace$. De la que más adelante, probaremos su convergencia.

La definición formal se presenta a continuación.

Definición. Se dice que una sucesión $\{a_n \}$ de números reales está acotada si existe un número real $M > 0$ tal que $|a_n| \leq M$ para todo $n \in \mathbb{N}.$

Proposición. Sea $\{ a_n \}$ una sucesión en $\mathbb{R}$. Si $\{ a_n \}$ es convergente, es decir, si existe $L \in \mathbb{R}$ tal que $$\lim_{n \to \infty} a_n = L,$$ entonces $\{ a_n \}$ está acotada. 

Demostración.

Sea $\varepsilon = 1$, como $\{ a_n \}$ converge, entonces existe $n_0 \in \mathbb{N}$ tal que para todo $n \geq n_0$ se tiene que $|a_n – L| < 1$. Además, sabemos que $|a_n| – |L| \leq |a_n – L | < 1$, de lo cual se sigue que

$$|a_n| < 1 + |L| \quad \forall n \geq n_0.$$ 

Notemos que hasta ahora la cota que tenemos es útil únicamente para $n \geq n_0$. Para los primeros $n_0 – 1$ elementos de la sucesión, consideremos $\hat{M} = max \{ |a_1|, |a_2|, …, |a_{n_0-1}| \}$. Así, la cota para toda nuestra sucesión será $M = max \{ \hat{M}, 1 + |L| \}$.

Si $1 \leq n \leq n_0 – 1$, entonces $$|a_n| \leq \hat{M} \leq M \Rightarrow |a_n| \leq M. \tag{1}$$

Por otro lado, si $n \geq n_0$, entonces $$|a_n| \leq 1 + |L| \leq M \Rightarrow |a_n| \leq M. \tag{2}$$

Por (1) y (2), se sigue que para todo $n \in \mathbb{N}$ se tiene que $|a_n| \leq M$. De lo que se concluye que la sucesión $\{a_n\}$ está acotada.

$\square$

Observación. Criterio de no convergencia: Dado que toda sucesión convergente está acotada, entonces, por contrapuesta, si una sucesión no está acotada no puede ser convergente.

Ahora que hemos probado la proposición anterior, podríamos preguntarnos si el regreso es cierto, es decir, ¿toda sucesión acotada converge? La respuesta es no y, de hecho, el contraejemplo lo revisamos en una entrada anterior: $\{ (-1)^n \}$. Se demostró que es una sucesión no convergente y está acotada por $1$.

Producto de sucesiones convergentes

Después de conocer el concepto de sucesión acotado, estamos listos para probar que el producto de sucesiones convergentes converge al producto de sus límites.

Proposición. Sean $\{a_n \}$ y $\{ b_n \}$ dos sucesiones tales que
$$\lim_{n \to \infty} a_n = L \text{ y } \lim_{n \to \infty} b_n = M.$$
Entonces se tiene
$$\lim_{n \to \infty} (a_n \cdot b_n) = L \cdot M.$$

Demostración.

Sea $\varepsilon > 0$.

Con la finalidad de factorizar y obtener expresiones en términos de las sucesiones individuales, sumaremos un cero, $a_nM-a_nM$, a la expresión $|a_n b_n – LM|$.

\begin{align*}
|a_n b_n – LM| & = |a_n b_n – a_n M + a_n M – LM| \\
& \leq |a_n (b_n – M)| + M (a_n – L)| \\
& = |a_n | |b_n – M| + |M| |a_n – L|.
\end{align*}

Además, como $\{a_n\}$ es convergente, entonces está acotada, es decir, existe $J > 0$ tal que para todo $n \in \mathbb{N}$, entonces $|a_n| \leq J$. De la expresión anterior se sigue

\begin{align*}
|a_n b_n – LM| & \leq |a_n | |b_n – M| + |M| |a_n – L| \\
& \leq J|b_n – M| + |M| |a_n – L| \text{, pues } J > 0. \\
\end{align*}

$$\therefore |a_n b_n – LM| \leq J|b_n – M| + |M| |a_n – L|. \tag{1}$$

Consideremos $K = max\{J, |M| \}$. De $(1)$ se sigue que

$$|a_n b_n – LM| \leq K|b_n – M| + K|a_n – L|. \tag{2}$$

Consideremos $\frac{\varepsilon}{2K} > 0$.

\begin{gather*}
\text{Como } \lim_{n \to \infty} a_n = L \text{, existe } n_1 \in \mathbb{N}, \text{ tal que } \forall n \geq n_1, |a_n-L| < \frac{\varepsilon}{2K}. \\

\text{Como } \lim_{n \to \infty} b_n = M \text{, existe } n_2 \in \mathbb{N}, \text{ tal que } \forall n \geq n_2, |b_n-L| < \frac{\varepsilon}{2K}.
\end{gather*}

Tomemos $n_0 = max\{n_1, n_2\}$. Si $n \geq n_0$, entonces $n \geq n_1$, $n \geq n_2$ y junto con $(2)$ se sigue que

\begin{align*}
|a_n b_n – LM| & \leq K|b_n – M| + K|a_n – L| \\
& < K \left( \frac{\varepsilon}{2K} \right) + K \left( \frac{\varepsilon}{2K} \right) \\
& = \varepsilon.
\end{align*}

$$\therefore |a_n b_n – LM| < \varepsilon.$$

$$\therefore \lim_{n \to \infty} (a_n \cdot b_n) = L \cdot M.$$

$\square$

A continuación, revisaremos un par de ejemplos donde se usan las propiedades que se demostraron en la entrada.

Ejemplo 1. Encuentra el siguiente límite $$\lim_{n \to \infty} -\frac{3}{n^2} + 1.$$

En la entrada anterior, probamos que $$\lim_{n \to \infty} \frac{1}{n} = 0.$$

Además, $$- \frac{3}{n^2}+1 = -3 \cdot \frac{1}{n} \cdot \frac{1}{n} + 1.$$

Empleando la expresión anterior y haciendo uso de las propiedades demostradas, tenemos

\begin{align*}
\lim_{n \to \infty} -\frac{3}{n^2} + 1 & = \lim_{n \to \infty} -3 \cdot \frac{1}{n} \cdot \frac{1}{n} + 1\\
& = -3 \cdot 0 \cdot 0 + 1\\
& = 1.
\end{align*}

$$\therefore \lim_{n \to \infty} -\frac{3}{n^2} + 1 = 1.$$

Ejemplo 2. Encuentra el siguiente límite $$\lim_{n \to \infty} 2n-2\sqrt{n^2+n}+1.$$

Factorizando la expresión anterior, obtenemos que
\begin{align*}
2n-2\sqrt{n^2+n}+1 & = (n+1) – 2 \sqrt{n}\sqrt{n+1}+ n \\
&= \left( \sqrt{n+1}-\sqrt{n}\right)^2 \\
& = \left( \sqrt{n+1}-\sqrt{n}\right) \left( \sqrt{n+1}-\sqrt{n}\right).
\end{align*}

En la entrada anterior, probamos que $$\lim_{n \to \infty} \left( \sqrt{n+1}-\sqrt{n}\right) = 0.$$

De esta forma, se tiene que

\begin{align*}
\lim_{n \to \infty} 2n-2\sqrt{n^2+n}+1 & = \lim_{n \to \infty} \left( \sqrt{n+1}-\sqrt{n}\right) \left( \sqrt{n+1}-\sqrt{n}\right) \\
& = 0 \cdot 0 \\
& = 0.
\end{align*}

$$\therefore \lim_{n \to \infty} 2n-2\sqrt{n^2+n}+1 = 0.$$

Más adelante…

En la siguiente entrada revisaremos el caso de la división entre sucesiones convergentes. Complementaremos nuestro estudio revisando una categoría especial de sucesiones llamadas monótonas, y probaremos diversas propiedades de las mismas.

Tarea moral

A continuación hay algunos ejercicios para que practiques los conceptos vistos en esta entrada. Te será de mucha utilidad intentarlos para entender más la teoría vista.

  • Prueba que si $\{a_n \}$ y $\{b_n \}$ son dos sucesiones tales $\{ a_n \}$ y $\{a_n+b_n\}$ convergen, entonces $\{b_n \}$ también converge.
  • Prueba que si $\{a_n \}$ y $\{b_n \}$ son dos sucesiones tales $\{ a_n \}$ converge a $L \neq 0$ y $\{a_n \cdot b_n\}$ converge, entonces $\{b_n \}$ también converge.
  • Usando las proposiciones demostradas en esta entrada, encuentra el límite de las siguientes sucesiones:
    • $\{ c \cdot \frac{1}{n} \}$, con $c \in \mathbb{R}.$
    • $\{ \frac{10}{n} – 7\}.$
    • $\{ \frac{2n^2-n}{n^2} \cdot \left( \frac{10}{n} – 7 \right) \}.$

Entradas relacionadas

Agradecimientos

Trabajo realizado con el apoyo del Programa UNAM-DGAPA-PAPIME PE104522 «Hacia una modalidad a distancia de la Licenciatura en Matemáticas de la FC-UNAM – Etapa 2»